Logical Reasoning Question Stems Flashcards

1
Q

Which one of the following most accurately expresses the main conclusion drawn in the argument?

A

Main Point

How well did you know this?
1
Not at all
2
3
4
5
Perfectly
2
Q

Which one of the following is most strongly supported by the information above?

A

Most Strongly Supported

How well did you know this?
1
Not at all
2
3
4
5
Perfectly
3
Q

The claim that some painters are superior to others in the execution of their artistic visions plays which one of the following roles in the argument?

A

Argument Part

How well did you know this?
1
Not at all
2
3
4
5
Perfectly
4
Q

The economist’s statements, if true, most strongly support which one of the following?

A

Most Strongly Supported

How well did you know this?
1
Not at all
2
3
4
5
Perfectly
5
Q

Which one of the following is best supported by the information above?

A

Most Strongly Supported

How well did you know this?
1
Not at all
2
3
4
5
Perfectly
6
Q

Of the following, which one, if true, is the logically strongest counter that Cassie can make to Melvin’s argument?

A

Weaken

How well did you know this?
1
Not at all
2
3
4
5
Perfectly
7
Q

Which one of the following most accurately expresses the overall conclusion of the argument?

A

Main Point

How well did you know this?
1
Not at all
2
3
4
5
Perfectly
8
Q

Which one of the following most accurately expresses the main conclusion drawn in Leslie’s argument?

A

Main Point

How well did you know this?
1
Not at all
2
3
4
5
Perfectly
9
Q

Which one of the following statements is most supported by the information above?

A

Most Strongly Supported

How well did you know this?
1
Not at all
2
3
4
5
Perfectly
10
Q

Which one of the following most precisely describes the role played in the scientist’s argument by the statement that chemical R did not cause cancer in laboratory rats?

A

Argument Part

How well did you know this?
1
Not at all
2
3
4
5
Perfectly
11
Q

Which one of the following, if true, most weakens the argument?

A

Weaken

How well did you know this?
1
Not at all
2
3
4
5
Perfectly
12
Q

If the shareholder’s statements are true, which one of the following is most strongly supported by them?

A

Most Strongly Supported

How well did you know this?
1
Not at all
2
3
4
5
Perfectly
13
Q

Which one of the following, if true, most seriously undermines the conclusion drawn in the argument above?

A

Weaken

How well did you know this?
1
Not at all
2
3
4
5
Perfectly
14
Q

Which one of the following would, if true, most weaken the psychologist’s argument?

A

Weaken

How well did you know this?
1
Not at all
2
3
4
5
Perfectly
15
Q

Which one of the following statements follows logically from the statements above?

A

Must be True

How well did you know this?
1
Not at all
2
3
4
5
Perfectly
16
Q

Which one of the following can be properly inferred from the ecologist’s statements?

A

Must be True

How well did you know this?
1
Not at all
2
3
4
5
Perfectly
17
Q

Which one of the following, if true, most weakens the argument?

A

Weaken

How well did you know this?
1
Not at all
2
3
4
5
Perfectly
18
Q

The lawyer’s conclusion follows logically if which one of the following is assumed?

A

Sufficient Assumption

How well did you know this?
1
Not at all
2
3
4
5
Perfectly
19
Q

Which one of the following, if true, most strengthens the argument?

A

Strengthen

20
Q

The conclusion of the psychiatrist’s argument is most strongly supported if which one of the following completes the argument?

A

Strengthen

21
Q

Which one of the following principles, if valid, most helps to justify the reasoning in the argument?

A

Pseudo Sufficient Assumption

22
Q

Which one of the following statements, if true, most weakens the reasoning above?

A

Weaken

23
Q

Which one of the following, if true, lends the most support to the psychologist’s conclusion?

A

Strengthen

24
Q

Which one of the following principles, if valid, most helps to justify the reasoning in the advice columnist’s argument?

A

Pseudo Sufficient Assumption

25
Q

Which one of the following, if assumed, allows the conclusion of the therapist’s argument to be properly inferred?

A

Sufficient Assumption

26
Q

The critic’s conclusion follows logically if which one of the following is assumed?

A

Sufficient Assumption

27
Q

Which one of the following statements, if true, most strengthens the argument?

A

Strengthen

28
Q

If the statements above are true, which one of the following is most strongly supported on the basis of them?

A

Most Strongly Supported

29
Q

Which one of the following, if true, adds the most support to the argument?

A

Strengthen

30
Q

Which one of the following principles, if valid, most helps to justify the real estate agent’s argumentation?

A

Pseudo Sufficient Assumption

31
Q

Which one of the following can be properly inferred from the information above?

A

Must be True

32
Q

The information above provides the most support for which one of the following propositions?

A

Most Strongly Supported

33
Q

Which one of the following, if true, would most weaken the argument in the newspaper article?

A

Weaken

34
Q

The statements above, if true, most strongly support which one of the following?

A

Most Strongly Supported

35
Q

Which one of the following most accurately expresses the conclusion drawn in the botanist’s argument?

A

Main Point

36
Q

Which one of the following is most strongly supported by the information above?

A

Most Strongly Supported

37
Q

Which one of the following, if true, most weakens the argument above?

A

Weaken

38
Q

The argument’s conclusion is properly drawn if which one of the following is assumed?

A

Sufficient Assumption

39
Q

Which one of the following principles most helps to justify the mathematics teacher’s argument?

A

Pseudo Sufficient Assumption

40
Q

The conclusion of the argument follows logically if which one of the following is assumed?

A

Sufficient Assumption

41
Q

Which one of the following, if true, most strengthens the economist’s reasoning?

A

Strengthen

42
Q

Which one of the following, if true, would most strengthen the archaeologist’s reasoning?

A

Strengthen

43
Q

Which one of the following most accurately describes the role played in the farmer’s argument by the proposition that farmers have to use greater and greater amounts of costly insecticides to control insect pests?

A

Argument Part

44
Q

Which one of the following most accurately expresses the main conclusion of the gardener’s argument?

A

Main Point

45
Q

Which one of the following, if true, most weakens the argument above?

A

Weaken

46
Q

Which one of the following principles, if valid, most helps to justify Marian’s argument?

A

Pseudo Sufficient Assumption